What is the surface area when a curve is rotated about the x-axis?

Click For Summary
The discussion focuses on calculating the surface area of the curve y=ex when rotated about the x-axis, using the formula Surface Area = 2∏a∫b x√(1+(dy/dx)²)dx. The original poster struggles with integrating the expression 2∏0∫1 x√(1+e²x)dx, finding that substitution and integration by parts lead to complications. A participant suggests that the variable x in the integral should be replaced with f(x) for clarity, as this aligns with the professor's examples. The conversation emphasizes the importance of correctly applying the surface area formula and clarifying the variable used in the integral. The thread concludes with a link to additional resources for further assistance.
californicate
Messages
12
Reaction score
0

Homework Statement


Obtain the surface area when the curve y=ex, 0≤x≤1, is rotated about the x-axis


Homework Equations


Surface Area = 2∏ab x√(1+(dy/dx)2)dx


The Attempt at a Solution


I started with the the equation, Surface Area = 2∏01 x√(1+e2x)dx. However, whichever way I try to integrate I end up getting stuck. By substitution, Nothing ends up working so that the integral becomes simpler, much less only according to one variable. By parts, I just end up with messier and messier integrals. How should I approach this problem?

Thanks!
 
Physics news on Phys.org
californicate said:

Homework Statement


Obtain the surface area when the curve y=ex, 0≤x≤1, is rotated about the x-axis


Homework Equations


Surface Area = 2∏ab x√(1+(dy/dx)2)dx


The Attempt at a Solution


I started with the the equation, Surface Area = 2∏01 x√(1+e2x)dx. However, whichever way I try to integrate I end up getting stuck. By substitution, Nothing ends up working so that the integral becomes simpler, much less only according to one variable. By parts, I just end up with messier and messier integrals. How should I approach this problem?

Thanks!

Hi californicate! Welcome to PF!

Your relevant equation doesn't look right to me. :)
 
So should the x inside the integral be replaced with an f(x)? That's the equation the prof gave in class, however in examples he switched back and forth between using f(x) and x. I'll ask about it next class.

Thanks!
 
Question: A clock's minute hand has length 4 and its hour hand has length 3. What is the distance between the tips at the moment when it is increasing most rapidly?(Putnam Exam Question) Answer: Making assumption that both the hands moves at constant angular velocities, the answer is ## \sqrt{7} .## But don't you think this assumption is somewhat doubtful and wrong?

Similar threads

Replies
2
Views
2K
Replies
11
Views
3K
  • · Replies 1 ·
Replies
1
Views
2K
Replies
4
Views
2K
  • · Replies 9 ·
Replies
9
Views
2K
  • · Replies 2 ·
Replies
2
Views
1K
  • · Replies 5 ·
Replies
5
Views
2K
  • · Replies 5 ·
Replies
5
Views
2K
  • · Replies 4 ·
Replies
4
Views
2K
  • · Replies 8 ·
Replies
8
Views
3K